What is Angular momentum: Definition and 1000 Discussions

In physics, angular momentum (rarely, moment of momentum or rotational momentum) is the rotational equivalent of linear momentum. It is an important quantity in physics because it is a conserved quantity—the total angular momentum of a closed system remains constant.
In three dimensions, the angular momentum for a point particle is a pseudovector r × p, the cross product of the particle's position vector r (relative to some origin) and its momentum vector; the latter is p = mv in Newtonian mechanics. Unlike momentum, angular momentum depends on where the origin is chosen, since the particle's position is measured from it.
Just as for angular velocity, there are two special types of angular momentum of an object: the spin angular momentum is the angular momentum about the object's centre of mass, while the orbital angular momentum is the angular momentum about a chosen center of rotation. The total angular momentum is the sum of the spin and orbital angular momenta. The orbital angular momentum vector of a point particle is always parallel and directly proportional to its orbital angular velocity vector ω, where the constant of proportionality depends on both the mass of the particle and its distance from origin. The spin angular momentum vector of a rigid body is proportional but not always parallel to the spin angular velocity vector Ω, making the constant of proportionality a second-rank tensor rather than a scalar.
Angular momentum is an extensive quantity; i.e. the total angular momentum of any composite system is the sum of the angular momenta of its constituent parts. For a continuous rigid body or a fluid the total angular momentum is the volume integral of angular momentum density (i.e. angular momentum per unit volume in the limit as volume shrinks to zero) over the entire body.
Torque can be defined as the rate of change of angular momentum, analogous to force. The net external torque on any system is always equal to the total torque on the system; in other words, the sum of all internal torques of any system is always 0 (this is the rotational analogue of Newton's Third Law). Therefore, for a closed system (where there is no net external torque), the total torque on the system must be 0, which means that the total angular momentum of the system is constant. The conservation of angular momentum helps explain many observed phenomena, for example the increase in rotational speed of a spinning figure skater as the skater's arms are contracted, the high rotational rates of neutron stars, the Coriolis effect, and the precession of gyroscopes. In general, conservation limits the possible motion of a system but does not uniquely determine it.
In quantum mechanics, angular momentum (like other quantities) is expressed as an operator, and its one-dimensional projections have quantized eigenvalues. Angular momentum is subject to the Heisenberg uncertainty principle, implying that at any time, only one projection (also called "component") can be measured with definite precision; the other two then remain uncertain. Because of this, the axis of rotation of a quantum particle is undefined. Quantum particles do possess a type of non-orbital angular momentum called "spin", but this angular momentum does not correspond to a spinning motion.

View More On Wikipedia.org
  1. mjmnr3

    Why does a symmetric wavefunction imply the angular momentum is even?

    I looked in the instructor solutions, which are given by: But I don't quite understand the solution, so I hope you can help me understand it. First. Why do we even know we are working with wavefunctions with the quantum numbers n,l,m? Don't we only get these quantum numbers if the particles...
  2. J

    I Rotation rates of planets seem odd?

    Ok, I know there are a lot of strange things in our solar system. Can anyone explain why the small planets spin so slowly? and why does Jupiter spin so quickly? It seems like a ball of debris, getting smaller and smaller, would increase its speed like an ice-skater pulling their arms in...
  3. A

    Ball hits a pivoting rod, what torque changes the angular momentum of the ball?

    Okay, i know that as a ball collides with a pivoting rod on an axis, the ball has angular momentum. Therefore after the collision, the ball is stopped or slowed, and the rod swings. The ball provides a force and torque to the rod. But if I isolate the ball, isn't the only thing acting on the...
  4. F

    Finding angular Momentum from a force

    Is it correct to say that that τ=0 since r has the same directacion as F?? and for \vec{L} que need to find \vec{p} So I thought solving this dif equation ## \int dp/dt =−kq/r^2 +β^4/r^5## Do you agree in the path I am going?
  5. F

    Torque and angular momentum with a central force

    HI τ= r ˆr x - ##k / r ^ 2## ˆr= 0 right? since ˆr x ˆr is zero What about L?
  6. P

    To find the angular momentum of a disc

    I was first wondering wether we can solve this question by applying conservation or energy or not but after googling it I found that we can't apply conservation of energy since there will be some energy lost in this case. I don't know how this energy is getting lost. My second doubt was if we...
  7. E

    Work & energy VS conservation of angular momentum

    Summary:: Would energy method give us a different answer from conservation of angular momentum? Hello, I do not know how to type equations here. So, I typed my question in Word and attached it here. Please see photos. Note: This question is not a homework. I did not find it in textbooks or...
  8. H

    I The reference frame for angular momentum components

    In which coordinate system the components of angular momentum are quantized? Better to say, if we can select the coordinate system arbitrarily, how the components of angular momentum, say z-component, are always ##L_z=m\hbar##?
  9. Simon Bridge

    A Conservation of angular momentum in positron-electron annihilation

    Pretty much in a nutshell... fielded a question about how spin affects electron positron annihilation... ie do the spins have to be opposite in order to conserve angular momentum for two-photon annihilation to happen? Intuitively I figured that looks reasonable ... but decided to check, and...
  10. Saptarshi Sarkar

    Conservation of angular momentum and rotational kinetic energy

    I first tried to get the solution by conserving the rotational kinetic energy and got ##\omega'=\frac2{\sqrt5} \omega##. But, it was not the correct answer. Next I tried by conserving the angular momentum and got ##\omega'=\frac 45 \omega##, which is the correct answer. Why is the rotational...
  11. bob012345

    I Meaning of the Orbital Angular Momentum of Super Chiral Light

    In this article it discusses the generation of something called super chiral light and claims with metamaterials they can make it have very high angular momentum like l=100. What does that really mean? How does that relate in magnitude to the normally computed linear momentum of a photon p=h/λ...
  12. B

    Engineering Gyroscopic Precession - Mass of wheel & Angular Momentum

    Hello, I have this i am learning. I have been trying to find information online but have struggled to find anything which helps me. YouTube usually has good videos, but doesn't seem to on this. This is one topic i have never learned before. But keen to. I was hoping someone could help me...
  13. LCSphysicist

    Collision, angular momentum and energy

    IS my solution right? Comparing with the other solutions, the answer just exchange the signals, i don't know why, THats what ifound. And here is the three equations: {i use the point which occurs the collision} Lo = Lf >> 0 = Iw + M*Vcm(block) Eg = ct> mvo² = mvf² + MVcm² + Iw² I = ml²/3...
  14. E

    A Kerr Black Hole Angular Momentum Limit

    The Schwarzschild metric seems to model, for example, the earth’s gravity field above the earth’s surface pretty well, even though the Earth is not really a golf-ball sized black hole down at the center. Can the same be said for the Kerr metric? Does it model a rotating extended body’s gravity...
  15. M

    Conservation of angular momentum and its counterpart for linear momentum

    Hi, I have just joined the forum. Thank you all for being a part of such places so that people like me can get answers to the questions on their minds! --------------------------- I have been trying to understand how a quadcopter yaws. Referring to the figure below which is bird's eye view of...
  16. LCSphysicist

    Understanding Angular Momentum and Energy Equations

    Since the equations are, actually, the question, i will post the image with relevant equations here: it seems strange, I'm almost sure that I didn't make a mistake in the differentiation, but differentiating 9.8b I found 9.7a with both positive terms
  17. LCSphysicist

    Is there a proof about angular momentum conservation?

    Angular momentum can be exchanged between objects in a closed system, but total angular momentum before and after an exchange remains constant (is conserved). There is a proof about this conservation?
  18. LCSphysicist

    Angular momentum of a rotating disc

    "A smooth horizontal disc rotates with a constant angular velocity ω about a stationary vertical axis passing through its centre, the point O. At a moment t=0 a disc is set in motion from that point with velocity v0. Find the angular momentum M(t) of the disc relative to the point O in the...
  19. M

    Finding Angular Momentum Along x-Axis for t given z(0) = 0, ˙z(0)=0

    given z(0) = 0 as well as ˙z(0)=0 How would one find the angular momentum along the x-axis in terms of t. Currently, I have formulated the following: $${\ddot{z} = \frac{g}{1+(\frac{4R}{s})^2}}$$
  20. D

    2-body problem - conservation of angular momentum

    Hi With the 2-body problem relating to planetary orbits i have encountered the following ; the gravitational force on the reduced mass acts towards the large mass(Sun) and since it is a central force it exerts no torque about the fixed centre(Sun) so angular momentum is conserved. Conservation...
  21. R

    Understanding Kinetic Energy, Angular Momentum & Torque

    Well I am pretty sure that the kinetic energy stays the same because in this case the velocity vector and energy make a ninety degree angle so no work is done, but I am lost about angular momentum. It could decrease maybe if the torque is clockwise while the ship is going in a counterclockwise...
  22. cpgp

    Why is angular momentum conserved here?

    A cylinder of radius R spins with angular velocity w_0 . When the cylinder is gently laid on a plane, it skids for a short time and eventually rolls without slipping. What is the final angular velocity, w_f? The solution follows from angular momentum conservation. $$L_i = I \omega_0 = L_f =...
  23. Eggue

    Angular momentum of a system of a rotating rod and sliding rings

    I got the correct answer for the first part but I'm not sure why the answer for (b) is the same for (a). Wouldn't the rings falling off mean that I_f = \frac{1}{12}M_L L^2 only where I_F, M_L, L are the final moment of inertia, mass of the rod and length of the rod as opposed to I_f =...
  24. E

    Is angular momentum taken about a point or an axis?

    One part of König's theorem states that ##\vec{L} = \vec{L}_{\text{COM}} + \vec{L}^{'}##. The term ##\vec{L}^{'}## simply refers to the angular momentum wrt. the centre of mass. This is just a point, and doesn't have an axis implicitly associated with it (we have infinitely many choices!). The...
  25. D

    Conservation of linear and angular momentum

    Hi ; I have a few question regarding the conservation of linear and angular momentum. Would appreciate any help. 1 - When no external forces act are both linear and angular momentum conserved in all 3 directions separately or just the total linear/angular momentum conserved ? 2 - if I approach...
  26. JD_PM

    A Lorentz Transformations and Angular momentum | Tong's QFT notes

    I am reading Tong's lecture notes and I found an example in which there are several aspects I do not understand. This example is aimed at: - Understanding what is the analogy in field theory to the fact that, in classical mechanics, rotational invariance gives rise to conservation of angular...
  27. A

    I What's the importance of the squared of the angular momentum?

    In quantum mechanics one sees what J^2 can offer but why do we even consider looking at the eigenstates and eigenvalues of J^2 and a component of J, say J_z? Why don't we just use J?
  28. K

    I Decoupling of angular momentum

    Hello! I am reading some papers and I often noticed that it is mentioned that a strong magnetic field is able to decouple certain angular momenta from each other. For example in this paper: https://journals.aps.org/prl/pdf/10.1103/PhysRevLett.100.023003 they present a Hamiltonian (second column...
  29. LarryS

    Classical EM vs Orbital Angular Momentum

    Consider the following experiment from the point-of-view of classical mechanics and classical electromagnetism: An originally free electron then passes through a magnetic field that is oriented so that it causes the electron to turn to, say, the right. During the “turning” of the electron (a...
  30. Like Tony Stark

    Angular Momentum Conservation in Spacecraft Orbits

    Tell me if I'm right: A) Angular momentum is conserved because there are no external torques. Linear momentum isn't conserved because gravity is acting on the spacecraft . Mechanical energy isn't conserved because it has to change between different orbits. B) Parabolic orbit...
  31. V

    Moment of inertia, conservation of angular momentum and energy

    When I solved the problem using the conservation of angular momentum, I have got the correct result (ω = 0.006 rad/s). However, when I tried to find the answer using the conservation of energy the result was incorrect and I do not understand why.
  32. Physics4Funn

    A Can Photon Have Orbital Angular Momentum?

    This is a very special case. In my 50 years studying physics I have never seen any discussion of photons having orbital angular momentum. Any angular momentum for photons in orbit around a black hole must be a GR question. I have not specialized in GR but I don’t recall any discussion of it. I...
  33. J

    Angular momentum of two particles connected by a rigid bar

    Lets do it for the left (the right will be similar): ##r_{left}=[(L-a\sin\theta)\sin\phi,(L+a\cos\theta)\cos\phi]## so ##v_{left}=[-a\dot{\theta}\cos\theta\sin\phi+(L-a\sin\theta)\dot{\phi}\cos\phi,-a\dot{\theta}\sin\theta\cos\phi-(L+a\cos\theta)\dot{\phi}\sin\phi]##. Is this right?
  34. ozcliff

    Conversion of angular momentum to linear momentum

    The balls used in the game of lawn bowls are biased so that they travel in a curved path of decreasing radius. When a bowl in motion collides at a glancing angle with another bowl at rest, it -appears- to increase its velocity. Due to conservation of linear momentum the post-collision velocity...
  35. S

    Number of Angular Momentum States (3 particles)

    I can solve the two particle system easily enough: Using ##j_1 = 1## and ##j_2 = 1##, the possible total angular momentum values are ##j = 2, 1, 0##. With ## m = -j , -j+1, ..., j ##, ##j = 2: m = 2, 1, 0, -1, -2 ## (5 states) ##j = 1: m = 1, 0, -1## (3 states) ## j = 0: m = 0 ## (1 state) I...
  36. T

    Angular Momentum Vector and Torque Vector

    In studying gyroscopic progression, the angular momentum vector is added to the torque vector. As intuitively these two vectors seem to be qualitatively quite different, how do we know that both vectors are in the same vector field and that they can be manipulated using the rules of vector...
  37. barryj

    Linear and angular momentum problem: Ball hitting a rod

    Homework Statement:: Ball of mass mb and velocity vb hits rod of length L , Rod pivots about the center. What is the angular momentum aafter impact? Homework Equations:: I = 1/12 (mR^2) I = mR^2 See the attached figure. I understand the concept of linear and angular momentum separately but I...
  38. benny1993

    Conservation of Angular Momentum

    Suppose I have a system of two disks (identical in mass and size) one is fixed to a shaft at it's center point and rotating due to an external torque that's removed as soon as the rotational motion begins. The second disk is dropped from rest over the rotating disk and sticks together to the...
  39. I

    Expectation value of an angular momentum with a complex exponent

    I am struggling to figure out how to calculate the expectation value because I am finding it hard to do something with the exponential. I tried using Euler's formula and some commutator relations, but I am always left with some term like ##\exp(L_z)## that I am not sure how to get rid of.
  40. jisbon

    Conservation of angular momentum?

    Not sure what to do here, except using the conversation of angular momentum. Even then, is angular momentum conserved in this case even after attaching an external object here? Else, what laws can I use to solve this problem? Using conversation of angular momentum: $$\dfrac...
  41. Comeback City

    I Angular Momentum in a Solar Nebula

    Hello all! Hope everyone's been doing well! My question relates to the nebular theory of solar system formation. It is generally accepted that via the nebular hypothesis, matter in a nebula contracts on its own gravity and begins to spin, but I'm having trouble understanding why it must begin...
  42. A

    Angular momentum of a baseball

    k̂ direction = 0 kg*m^2/s ĵ direction = 0 kg*m^2/s î direction = (0.145kg) (20m/s) (6m) = 17.4 kg*m^2/s
  43. A

    Angular momentum of a falling ball

    (L) = (radius) * (mass*velocity) velocity= 0+ (9.8m/s^2) (0.7s) = 6.86m/s (L) = (0.77m) * (2kg*6.86m/s)= 1.05 kg*m^2/s angular momentum points towards Polly
  44. H

    I Matrix Representation of the Angular Momentum Raising Operator

    In calculating the matrix elements for the raising operator L(+) with l = 1 and m = -1, 0, 1 each of my elements conforms to a diagonal shifted over one column with values [(2)^1/2]hbar on that diagonal, except for the element, L(+)|0,-1>, where I have a problem. This should be value...
  45. E

    Does an impulse contribute to both linear and angular momentum?

    As an analogue, if 5J of work is done on an object then the linear KE might increase by 2J and the angular by 3J, so the work is divided between the linear and rotational forms. Now suppose there is a sphere sliding on a frictionless surface. If an impulse of magnitude 1Ns is applied to the...
  46. Santilopez10

    Angular momentum of a mass-rope-mass system

    1) the motion equations for ##m_2## are: $$T-m_2 g=0 \rightarrow T=m_2 g$$ ##m_1##: $$T=m_1\frac{v^2}{r_0} \rightarrow \vec {v_0}=\sqrt{\frac{r_0 g m_2}{m_1}}\hat{\theta}$$ 2) This is where I am stuck, first I wrote ##m_2## motion equation just like before, but in polar coordinates...
Back
Top